On the integrals $\int_0^\infty [1-x^n \operatorname{arccot}^n(x)] \, \mathrm{d} x$

asymptoticscalculusdefinite integralsintegration

When answering this question I came across the integrals

$$ I_n \equiv \int \limits_0^\infty [1-x^n \operatorname{arccot}^n(x)] \, \mathrm{d} x \, , ~n \in \mathbb{N} \, . $$

I needed $I_1 = \frac{\pi}{4}$ , $I_2 = \frac{\pi}{6}[2 \ln(2)+1]$ and $I_3 = \frac{\pi}{32}[32 \ln(2)+ 4 -\pi^2]$ in my answers. They can be evaluated by writing
$$ I_n = \lim_{r \to \infty} \left[r – \int \limits_0^r x^n \operatorname{arccot}^n (x) \, \mathrm{d} x \right] $$
and using repeated integration by parts to reduce the remaining integral to a few terms that cancel the $r$ in the limit and some well-known integrals.

This calculation should work for any $n \in \mathbb{N}$, but it gets more tedious for larger values of course. Mathematica gives reasonably nice expressions for the integrals in terms of $\pi$ , $\ln(2)$ and values of the zeta function (for example $I_4 = \frac{\pi}{40} [4 + 80 \ln(2) – \pi^2 (3 + 4 \ln(2)) + 18 \zeta(3)]$), so it might be possible to evaluate the integrals in terms of suitable special functions in general.

However, I have not yet found a way to transform them into such an expression. The obvious substitution $x = \cot (t)$ leads to
$$ I_n = \int \limits_0^{\pi/2} \frac{\sin^n (t) – t^n \cos^n (t)}{\sin^{n+2}(t)} \, \mathrm{d} t \, ,$$
which does not seem to help much. We can use
$$ 1 – a^n = (1-a) \sum \limits_{k=0}^{n-1} a^k = \sum \limits_{k=0}^{n-1} \sum \limits_{l=0}^k (-1)^l {k \choose l} (1-a)^{l+1} $$
for $n \in \mathbb{N}$ and $a \in \mathbb{R}$ to obtain
$$ I_n = \sum \limits_{k=0}^{n-1} \sum \limits_{l=0}^k (-1)^l {k \choose l} J_{l+1} $$
in terms of
$$ J_n \equiv \int \limits_0^\infty [1-x \operatorname{arccot}(x)]^n \, \mathrm{d} x \, , ~n \in \mathbb{N} \, . $$
Interchanging the sums, using $\sum_{k=l}^{n-1} {k \choose l} = {n \choose l+1}$ (which apparently is known as the hockey-stick identity) and defining $I_0 = J_0 = 0$ , we find that the two sequences are binomial transforms of each other (except for a minus sign):
$$I_n = – \sum \limits_{m=0}^n (-1)^m {n \choose m} J_m \, , \, n \in \mathbb{N}_0 \, . $$
I do not know which of the two families of integrals is easier to evaluate though. Note that the same method enables us to compute
$$ \int \limits_0^\infty [1-x \operatorname{arccot}(x)] P[x \operatorname{arccot}(x)] \, \mathrm{d} x $$
for any polynomial $P$ once we know $(I_n)_{n \in \mathbb{N}}$ or $(J_n)_{n \in \mathbb{N}}$.

I am also interested in the asymptotic behaviour of the integrals. Numerical calculations and plots suggest that we have
\begin{align}
I_n &\sim \sqrt{\frac{\pi n}{3}} \, , \, n \to \infty \, , \\
J_n &\sim \frac{2}{\pi n} \, , \, n \to \infty \, ,
\end{align}
but I have no idea how to prove that.

Therefore I am left with the following two questions:

  1. How can we find a closed-form expression for $I_n$ or $J_n$ , $n \in \mathbb{N}$ ?
  2. What can we say about the asymptotics of $I_n$ or $J_n$ as $n \to \infty$ ?

Any hints or (partial) solutions to either of them would be greatly appreciated.

Best Answer

Cool problem. Both proposed asymptotic forms are correct. I'm only going to do a first order calc for both, and for $J_n$ I'll say where the proof needs some work. For $I_n$ use the expression given by Arathoon's first note. Observe that $t\cot(t)$ is unimodal decreasing and at $t=0$ has the value 1. Thus by raising it to a high power we expect it to become more sharply peaked. In fact it is gaussian because $$ t\cot(t) = 1-t^2/3-t^4/45... \sim \exp(-t^2/3)(1-7/90t^4 + ...) $$ Therefore $$I_n = \int_0^{\pi/2} \frac{dt}{\sin^2t} \big(1-(t\cot(t))^n \big) \sim \int_0^{\pi/2} \frac{dt}{\sin^2t} \big(1-\exp{(-n\,t^2/3)} \big).$$ Because $n \to \infty$ and because of the $\csc^2t,$ it is seen that the most important region is near $t=0.$ The trick now is to recognize that $\tan^2t = t^2 + O(t^4).$ Replace $t^2$ in the exponential with $\tan^2t$ because it just so happens (Mathematica knows it too) that $$ \int_0^{\pi/2} \frac{dt}{\sin^2t} \big(1-\exp{(-a\,\tan^2t)} \big)=\sqrt{a \pi}.$$ With $a=n/3,$ we indeed have $I_n \sim \sqrt{n \pi /3}.$

For $J_n$ I used a technique call $\textit{depoissonization}.$ Make an exponential power series, $$\sum_{n=0}^\infty \frac{y^n}{n!}J_n = \sum_{n=0}^\infty\frac{y^n}{n!} \int_0^{\infty}(1-t\cot^{-1}(t))^n\, dt = e^y\int_0^{\infty}\exp{(-y\,t\,\cot^{-1}{t} )} \, dt. $$ Since $t\cot^{-1}t$ is monotonically increasing, all we need is the behavior near $0$ to asymptotically estimate the integral. In fact, $t\cot^{-1}t = \pi\,t/2 +O(t^2).$ Using the first term we therefore find $$J(y) := e^{-y}\sum_{n=0}^\infty \frac{y^n}{n!}J_n \sim \int_0^{\infty} \exp{(-\pi\,y\,t/2)} \, dt = \frac{2}{\pi\,y}.$$ By depoissonization, $J_n \sim J(n) =2/(\pi n)$ as long as the next term in the sequence is smaller than this first term. I don't intend to show that. One thing that makes this problem interesting is that in fact it can be shown that $I_n = \sum_{k=1}^n(-1)^{k+1}\binom{n}{k}J_k$ so that $I_n$ and $J_n$ are binomial transforms. Putting in the first order asymptotic for for either $I_n$ or $J_n$ will $\textit{not}$ give you the asymptotics of its transform.